57.2.25 The law of the city

Accounts PlayableAccounts Playable Live Sage
edited November 2015 in Logical Reasoning 3107 karma
EDIT: I had the wrong question in the title

I have a huge bone to pick with this one. I don't see how any of the answer choices "must be true."

Here is the law:
Greater than $100 AND made by nonresident AND nonresident isn't former resident--->Register

The campaign complied with the law (so it complied with the contrapositive as well). The campaign accepted contributions only from residents and former residents.

What I am looking for: I thought this was a pretty weird passage since the final clause only denies the sufficient condition, which tells us nothing about if those contributions needed to be registered. Thus, I thought an answer choice was going to specifically reaffirm this premise.

Answer A: This is what I chose, even though I was confident it was wrong. I chose it since I thought all of the answer choices didn't work. This answer is incorrect because the dollar amount of the people that donated to the campaign is irrelevant. The nonresident in this answer choice MUST have been former resident, and if this were what this had said, it would have been correct. However, the dollar amount could have been anything since our conditional rule is irrelevant.

Answer B: This could be true, but it doesn't have to be true. We only know one sufficient condition for registration; there could be multiple sufficient conditions for registration.

Answer C: This is the correct answer??? How must this be true? We only know one rule/sufficient condition for registration. Why can't there be other rules? The passage never indicates that the rule given is the only rule. What if all contributions from residents and former residents must be registered? How this is even close to being a correct answer is beyond my understanding.

Answer D: Again, this doesn't have to be true. We know nothing about donations by the residents and former residents. They each could have given the campaign $1 or something.

Answer E: Again, we only have one sufficiency condition for registration, so this could be true.

Comments

  • nicole.hopkinsnicole.hopkins Inactive Sage Inactive ⭐
    edited November 2015 7965 karma
    @"Accounts Playable" said:
    This is what I chose, even though I was confident it was wrong.
    Just a quick thought ... Given that your ONE job on this test is to eliminate wrong answer choices, why would you choose one that you could eliminate (I'm assuming you had reasons to do so given your confidence that it was wrong)?
  • Accounts PlayableAccounts Playable Live Sage
    3107 karma
    @nicole.hopkins

    Time was ticking and I had question 26 to still do. I eliminated all of the answer choices, so I had to just pick one and move on.
Sign In or Register to comment.